|
10 | 10 | \large
|
11 | 11 | \setlength{\baselineskip}{1.2em}
|
12 | 12 | \ifpreface
|
13 |
| - \input{../../../global/preface} |
14 |
| - \newgeometry{left=2cm,right=2cm,top=2cm,bottom=2cm} |
| 13 | + \input{../../../global/preface} |
| 14 | + \newgeometry{left=2cm,right=2cm,top=2cm,bottom=2cm} |
15 | 15 | \else
|
16 |
| - \newgeometry{left=2cm,right=2cm,top=2cm,bottom=2cm} |
17 |
| - \maketitle |
| 16 | + \newgeometry{left=2cm,right=2cm,top=2cm,bottom=2cm} |
| 17 | + \maketitle |
18 | 18 | \fi
|
19 | 19 | %from_here_to_type
|
20 | 20 | \newcommand{\A}{\mathbbm{A}}
|
21 | 21 | \begin{problem}
|
22 |
| - $P$ is an ideal of a unitary commutative ring $A$, then $P$ is prime ideal of $A\iff A/P$ is integral domain. |
| 22 | +$P$ is an ideal of a unitary commutative ring $A$, then $P$ is prime ideal of $A\iff A/P$ is integral domain. |
23 | 23 | \end{problem}
|
24 | 24 | \(\)
|
25 | 25 | \begin{solution}
|
26 |
| - $\Rightarrow $: |
| 26 | + $\Rightarrow $: |
27 | 27 |
|
28 |
| - Since $A$ is a unitary commutative ring, so $A/P$ is unitary commutative ring, too. So we only need to prove $[ab]=[0]\Rightarrow [a]=[0]\xor [b]=[0]$. |
29 |
| - Obviously $[ab]=0\iff ab\in P\iff a\in P\xor b\in P\iff [a]=[0]\xor [b]=[0]$. |
| 28 | + Since $A$ is a unitary commutative ring, so $A/P$ is unitary commutative ring, too. So we only need to prove $[ab]=[0]\Rightarrow [a]=[0]\xor [b]=[0]$. |
| 29 | + Obviously $[ab]=0\iff ab\in P\iff a\in P\xor b\in P\iff [a]=[0]\xor [b]=[0]$. |
30 | 30 |
|
31 |
| - $\Leftarrow $: |
| 31 | + $\Leftarrow $: |
32 | 32 |
|
33 |
| - As the same, $ab\in P\iff [ab]=[0]\Rightarrow [a]=[0]\xor [b]=[0]\iff a\in P\xor b\in P$, so $P$ is prime ideal. |
| 33 | + As the same, $ab\in P\iff [ab]=[0]\Rightarrow [a]=[0]\xor [b]=[0]\iff a\in P\xor b\in P$, so $P$ is prime ideal. |
34 | 34 | \end{solution}
|
35 | 35 |
|
36 | 36 | \begin{problem}
|
37 |
| - $M$ is an ideal of a unitary commutative ring $A$, then $M$ is maximal ideal of $A \iff A/M$ is a field. |
| 37 | +$M$ is an ideal of a unitary commutative ring $A$, then $M$ is maximal ideal of $A \iff A/M$ is a field. |
38 | 38 | \end{problem}
|
39 | 39 | \begin{solution}
|
40 |
| - $\Rightarrow $: |
| 40 | + $\Rightarrow $: |
41 | 41 |
|
42 |
| - Consider $[a]\in A/M\minus [0]$, we will prove it has a reverse. Consider $N:=\{xm+ya:x,y\in A,m\in M\}$ is the minimum ideal of $A$ contains $M$ and $a$. |
43 |
| - Since $[a]\neq [0]$ we know $a\notin M$, so $M\subsetneqq N$. |
44 |
| - Noting $M$ is maximal, so $N=A$. That means $\exists x,y\in A,m\in M,xm+ya=1$. So $[xm+ya]=[1]$. Since $[xm]=[0]$ we get $[y][a]=1$, i.e., $[y]=[a]^{-1}$. |
| 42 | + Consider $[a]\in A/M\minus [0]$, we will prove it has a reverse. Consider $N:=\{xm+ya:x,y\in A,m\in M\}$ is the minimum ideal of $A$ contains $M$ and $a$. |
| 43 | + Since $[a]\neq [0]$ we know $a\notin M$, so $M\subsetneqq N$. |
| 44 | + Noting $M$ is maximal, so $N=A$. That means $\exists x,y\in A,m\in M,xm+ya=1$. So $[xm+ya]=[1]$. Since $[xm]=[0]$ we get $[y][a]=1$, i.e., $[y]=[a]^{-1}$. |
45 | 45 |
|
46 |
| - $\Leftarrow $: |
| 46 | + $\Leftarrow $: |
47 | 47 |
|
48 |
| - Consider $a\in A\minus M, N:=\{xp+ya:x,y\in A,p\in P\}$, we will prove $N=A$, which means $M$ is maximal. |
49 |
| - Since $A/M$ is field, $\exists y\in A, [y]=[a]^{-1}$. That's means $ay-1\in M\subset N$. Noting $ay\in N$, so $1\in N$, thus $N=A$. |
| 48 | + Consider $a\in A\setminus M, N:=\{xp+ya:x,y\in A,p\in P\}$, we will prove $N=A$, which means $M$ is maximal. |
| 49 | + Since $A/M$ is field, $\exists y\in A, [y]=[a]^{-1}$. That's means $ay-1\in M\subset N$. Noting $ay\in N$, so $1\in N$, thus $N=A$. |
50 | 50 | \end{solution}
|
51 | 51 |
|
52 | 52 | \begin{problem}
|
53 |
| - A ring $A$ is noetherian, $I\subset A$ is an ideal of $A$, then $A/I$ is noetherian. |
| 53 | +A ring $A$ is noetherian, $I\subset A$ is an ideal of $A$, then $A/I$ is noetherian. |
54 | 54 | \end{problem}
|
55 | 55 | \begin{solution}
|
56 |
| - Consider an ideal $J\subset A/I$, let $M:=\{x\in A:[x]\in J\}$. Then $\forall a\in A,x\in M,[ax]=[a][x]\in J$, so $ax\in M$. $\forall a,b\in M,[a-b]=[a]-[b]\in J$, so $a-b\in M$. So $M$ is an ideal of $A$. |
57 |
| - Since $A$ is noetherian, we can assume $M=(f_i,i=1,2,\cdots n)$. Now we will prove $J=([f_i],i=1,2,\cdots n)$. |
58 |
| - Consider $[f]\in J$, from definition of $M$ we know $f\in M$, so $f=\sum_{i=1}^na_if_i,a_i\in A$, thus $[f]=\left[\sum_{i=1}^na_if_i\right]=\sum_{i=1}^n[a_i][f_i]$. So $J=([f_i],i=1,2,\cdots n)$. |
| 56 | + Consider an ideal $J\subset A/I$, let $M:=\{x\in A:[x]\in J\}$. Then $\forall a\in A,x\in M,[ax]=[a][x]\in J$, so $ax\in M$. $\forall a,b\in M,[a-b]=[a]-[b]\in J$, so $a-b\in M$. So $M$ is an ideal of $A$. |
| 57 | + Since $A$ is noetherian, we can assume $M=(f_i,i=1,2,\cdots n)$. Now we will prove $J=([f_i],i=1,2,\cdots n)$. |
| 58 | + Consider $[f]\in J$, from definition of $M$ we know $f\in M$, so $f=\sum_{i=1}^na_if_i,a_i\in A$, thus $[f]=\left[\sum_{i=1}^na_if_i\right]=\sum_{i=1}^n[a_i][f_i]$. So $J=([f_i],i=1,2,\cdots n)$. |
59 | 59 | \end{solution}
|
60 | 60 |
|
61 | 61 | \begin{problem}
|
62 |
| - \label{pro:4} |
63 |
| - $K$ is a field, $A=K[x_1,x_2,\cdots x_n],\A^n_K=K^n$. For ideal $I$ of $A$ let $V(I):=\{p\in\A_K^n:f(p)=0,\forall f\in I\}$. Then $V(I_1)\cup V(I_2)=V(I_1\cap I_2)=V(I_1I_2)$ |
| 62 | +\label{pro:4} |
| 63 | +$K$ is a field, $A=K[x_1,x_2,\cdots x_n],\A^n_K=K^n$. For ideal $I$ of $A$ let $V(I):=\{p\in\A_K^n:f(p)=0,\forall f\in I\}$. Then $V(I_1)\cup V(I_2)=V(I_1\cap I_2)=V(I_1I_2)$ |
64 | 64 | \end{problem}
|
65 | 65 | \begin{solution}
|
66 |
| - \begin{lemma} |
67 |
| - \label{lem:1} |
68 |
| - $I\subset J\Rightarrow V(I)\supset V(J)$. |
69 |
| - \end{lemma} |
70 |
| - \begin{proof} |
71 |
| - Consider $p\in V(J)$, we get $\forall f\in J,f(p)=0$. Since $I\subset J$, so $\forall f\in I,f(p)=0$, i.e., $f\in V(I)$. |
72 |
| - \end{proof} |
73 |
| - From Lemma \Cref{lem:1} we know $V(I_1\cap I_2)\subset V(I_1I_2)$, so we only need to prove $V(I_1)\cup V(I_2)\subset V(I_1\cap I_2),V(I_1I_2)\subset V(I_1)\cup V(I_2)$. |
74 |
| - \begin{itemize} |
75 |
| - \item $V(I_1)\cup V(I_2)\subset V(I_1\cap I_2)$: |
76 |
| - From \Cref{lem:1} It's obvious. |
77 |
| - \item $V(I_1I_2)\subset V(I_1)\cup V(I_2)$: |
78 |
| - Consider $p\in V(I_1I_2)$. |
79 |
| - If $p\notin V(I_1)\cup V(I_2)$, then $\exists f_1\in I_1,f_2\in I_2, f_1(p)\neq 0,f_2(p)\neq 0$. Now consider $f=f_1f_2\in I_1I_2$, we get $f(p)=f_1(p)f_2(p)\neq 0$, so $p\notin V(I_1I_2)$, it's a contradiction. |
80 |
| - \end{itemize} |
| 66 | + \begin{lemma} |
| 67 | + \label{lem:1} |
| 68 | + $I\subset J\Rightarrow V(I)\supset V(J)$. |
| 69 | + \end{lemma} |
| 70 | + \begin{proof} |
| 71 | + Consider $p\in V(J)$, we get $\forall f\in J,f(p)=0$. Since $I\subset J$, so $\forall f\in I,f(p)=0$, i.e., $f\in V(I)$. |
| 72 | + \end{proof} |
| 73 | + From Lemma \Cref{lem:1} we know $V(I_1\cap I_2)\subset V(I_1I_2)$, so we only need to prove $V(I_1)\cup V(I_2)\subset V(I_1\cap I_2),V(I_1I_2)\subset V(I_1)\cup V(I_2)$. |
| 74 | + \begin{itemize} |
| 75 | + \item $V(I_1)\cup V(I_2)\subset V(I_1\cap I_2)$: |
| 76 | + From \Cref{lem:1} It's obvious. |
| 77 | + \item $V(I_1I_2)\subset V(I_1)\cup V(I_2)$: |
| 78 | + Consider $p\in V(I_1I_2)$. |
| 79 | + If $p\notin V(I_1)\cup V(I_2)$, then $\exists f_1\in I_1,f_2\in I_2, f_1(p)\neq 0,f_2(p)\neq 0$. Now consider $f=f_1f_2\in I_1I_2$, we get $f(p)=f_1(p)f_2(p)\neq 0$, so $p\notin V(I_1I_2)$, it's a contradiction. |
| 80 | + \end{itemize} |
81 | 81 | \end{solution}
|
82 | 82 |
|
83 | 83 | \begin{problem}
|
84 |
| - $K$ is an infinite field, then $\A_k^n$ is not Hausdorff. |
| 84 | +$K$ is an infinite field, then $\A_k^n$ is not Hausdorff. |
85 | 85 | \end{problem}
|
86 | 86 | \begin{solution}
|
87 |
| - \begin{lemma} |
88 |
| - \label{lem:2} |
89 |
| - $K$ is an infinite field, $f\in K[x_1,x_2,\cdots x_n]\setminus \{0\}$, then exists $p\in \A_K^n,f(p)\neq 0$. |
90 |
| - \end{lemma} |
91 |
| - \begin{proof} |
92 |
| - Use MI. When $n=0,K[x_1,x_2,\cdots x_n]=K$, so it's obvious. Assume for $n=k$ it's right, when goes to $k+1$: |
| 87 | + \begin{lemma} |
| 88 | + \label{lem:2} |
| 89 | + $K$ is an infinite field, $f\in K[x_1,x_2,\cdots x_n]\setminus \{0\}$, then exists $p\in \A_K^n,f(p)\neq 0$. |
| 90 | + \end{lemma} |
| 91 | + \begin{proof} |
| 92 | + Use MI. When $n=0,K[x_1,x_2,\cdots x_n]=K$, so it's obvious. Assume for $n=k$ it's right, when goes to $k+1$: |
93 | 93 |
|
94 |
| - Consider $h\in K(x_1,x_2,\cdots x_k)[x_{k+1}],h(x_{k+1}):=f(x_1,x_2,\cdots x_k,x_{k+1})$ is a non-zero polynomial so it has finite root. So exists $a\in K,h(a)\neq 0$. So $g:=f(x_1,x_2,\cdots x_k,a)\in K[x_1,x_2,\cdots x_k]\neq 0$. |
95 |
| - By induction hypothesis we get $\exists b\in \A_K^k,g(b)\neq 0$. Let $p:=(b,a)\in \A_K^{k+1}$, then $f(p)\neq 0$. |
96 |
| - \end{proof} |
97 |
| - In fact, it's not only not Hausdorff, it's kind of ''absolutly not Hausdorff'' because every pair of point can't be seperated. |
98 |
| - Consider two point $p\neq q,p,q\in \A_K^n$. |
99 |
| - Assume $p=(p_1,p_2,\cdots p_n),q=(q_1,q_2,\cdots q_n),p_1\neq q_1$. Assume two open set $V(I_1)^c,V(I_2)^c$ can seperate $p,q$, then $V(I_1)\cup V(I_2)=\A_K^n$. |
100 |
| - From \Cref{pro:4} we know $V(I_1I_2)=\A_K^n$. So $\forall f\in V(I_1I_2),\forall p\in \A_K^n,f(p)=0$. |
101 |
| - Then from Lemma\Cref{lem:2} we can get $f=0$. So $I_1I_2=\{0\}$. |
102 |
| - Since $p\notin V(I_1),q\notin V(I_2)$, we know $I_1,I_2\neq \{0\}$. So $\exists f_1\in I_1\neq 0,\exists f_2\in I_2\neq 0$, and thus $f=f_1f_2\in I_1I_2\neq 0$, contradiction! |
103 |
| - \(\phi\) |
104 |
| - So there is not a pair of points can be seperated by two disjoint open set. |
| 94 | + Consider $h\in K(x_1,x_2,\cdots x_k)[x_{k+1}],h(x_{k+1}):=f(x_1,x_2,\cdots x_k,x_{k+1})$ is a non-zero polynomial so it has finite root. So exists $a\in K,h(a)\neq 0$. So $g:=f(x_1,x_2,\cdots x_k,a)\in K[x_1,x_2,\cdots x_k]\neq 0$. |
| 95 | + By induction hypothesis we get $\exists b\in \A_K^k,g(b)\neq 0$. Let $p:=(b,a)\in \A_K^{k+1}$, then $f(p)\neq 0$. |
| 96 | + \end{proof} |
| 97 | + In fact, it's not only not Hausdorff, it's kind of ''absolutly not Hausdorff'' because every pair of point can't be seperated. |
| 98 | + Consider two point $p\neq q,p,q\in \A_K^n$. |
| 99 | + Assume $p=(p_1,p_2,\cdots p_n),q=(q_1,q_2,\cdots q_n),p_1\neq q_1$. Assume two open set $V(I_1)^c,V(I_2)^c$ can seperate $p,q$, then $V(I_1)\cup V(I_2)=\A_K^n$. |
| 100 | + From \Cref{pro:4} we know $V(I_1I_2)=\A_K^n$. So $\forall f\in V(I_1I_2),\forall p\in \A_K^n,f(p)=0$. |
| 101 | + Then from Lemma\Cref{lem:2} we can get $f=0$. So $I_1I_2=\{0\}$. |
| 102 | + Since $p\notin V(I_1),q\notin V(I_2)$, we know $I_1,I_2\neq \{0\}$. So $\exists f_1\in I_1\neq 0,\exists f_2\in I_2\neq 0$, and thus $f=f_1f_2\in I_1I_2\neq 0$, contradiction! |
| 103 | + \(\phi\) |
| 104 | + So there is not a pair of points can be seperated by two disjoint open set. |
105 | 105 | \end{solution}
|
106 | 106 | \begin{figure}
|
107 |
| - \centering |
108 |
| - \includegraphics{~/Pictures/2-3.jpg}% |
| 107 | + \centering |
| 108 | + \includegraphics{~/Pictures/2-3.jpg}% |
109 | 109 |
|
110 | 110 | \end{figure}
|
111 | 111 | \end{document}
|
|
0 commit comments